Why must q be the least element for (q+1)a to be greater than b?

Click For Summary
For natural numbers a and b, there exists a unique pair (q, r) such that b = aq + r, where 0 ≤ r < a. The discussion centers on why q must be the least integer for which (q + 1)a exceeds b. If q were not the least, then there would be a non-negative integer p ≤ q such that pa ≤ b, contradicting the definition of q. This leads to the conclusion that r must be less than a, confirming the uniqueness of the pair (q, r). The proof hinges on the properties of integers and their relationships in the context of division.
Mathematicsresear
Messages
66
Reaction score
0
Member asked to not delete the template in future posts

Homework Statement


Let a, b be natural numbers then there exists a unique pair (q,r) that are elements of the non-negative integers such that b=aq+r and 0 is less than or equal to r which is less than a

I have a question regarding the existence part of the proof, now if I assumed a is less than b, its clear that there exists a positive integer x such that xa is greater than b. Now, why must q be the least element such that (q+1)a is greater than b?
 
Physics news on Phys.org
Assume the opposite and show that in that case r > a
 
  • Like
Likes berkeman
Mathematicsresear said:

Homework Statement


Let a, b be natural numbers then there exists a unique pair (q,r) that are elements of the non-negative integers such that b=aq+r and 0 is less than or equal to r which is less than a

I have a question regarding the existence part of the proof, now if I assumed a is less than b, its clear that there exists a positive integer x such that xa is greater than b. Now, why must q be the least element such that (q+1)a is greater than b?

If ##q## is the least element such that ##(q+1)a > b## then for all non-negative integers ##p \leq q## we have ##pa \leq b.## In particular, ##qa \leq b## but ##(q+1) a## is not ##\leq b##. That means that ##b-qa \in \{0,1,\ldots, a-1 \}.##
 
Question: A clock's minute hand has length 4 and its hour hand has length 3. What is the distance between the tips at the moment when it is increasing most rapidly?(Putnam Exam Question) Answer: Making assumption that both the hands moves at constant angular velocities, the answer is ## \sqrt{7} .## But don't you think this assumption is somewhat doubtful and wrong?

Similar threads

Replies
15
Views
4K
  • · Replies 1 ·
Replies
1
Views
1K
  • · Replies 1 ·
Replies
1
Views
2K
  • · Replies 10 ·
Replies
10
Views
2K
  • · Replies 2 ·
Replies
2
Views
1K
Replies
11
Views
2K
  • · Replies 5 ·
Replies
5
Views
2K
Replies
3
Views
2K
  • · Replies 3 ·
Replies
3
Views
2K
  • · Replies 5 ·
Replies
5
Views
3K